2
$\begingroup$

Does anybody have access to the paper A. Kotzig: On well spread sets of integers, 1972, or does anybody know the proof of $\sigma^*(n)\geq 4+\binom{n-1}2$ for $n\geq7$ (as cited in Marr,Wallis: Magic Graphs, Springer 2013, Th. 2.15)

Notions: A set $A=\{a_1,\dots,a_n\}$ of integers is a well-spread (weak Sidon) set of cardinality $n$, if $a_i+a_j \neq a_k+a_\ell$ whenever these four elements of $A$ are distinct. Then $\sigma(A):= \max A - \min A +1$ is the size of $A$, and $\sigma^*(n)$ is the minimal $\sigma(A)$ taken over all well-spread sets $A$ of cardinality $n$.

$\endgroup$
3
  • $\begingroup$ Bibliographic info: A. Kotzig, On well spread sets of integers. Tech. Report CRM-161, Centre Res. Math., Université de Montréal (1972), 83 pp. – maybe it's worth contacting the U of Montreal. $\endgroup$ Mar 11, 2020 at 11:37
  • $\begingroup$ For $n=3$ we have a weak Sidon set of spread 3, which looks to be less than 5. $\endgroup$ Mar 11, 2020 at 15:05
  • $\begingroup$ The lower bound on $\sigma^*(n)$ holds for $n\geq7$. $\endgroup$
    – BerndM
    Mar 11, 2020 at 15:57

1 Answer 1

3
$\begingroup$

Suppose that $A\subset[1,l]$ is weak Sidon; we show that $l>\binom{n-1}2$, which is very close to your estimate.

There are $n(n-1)/2$ positive differences of the form $a_1-a_2$ with $a_1,a_2\in A$. All these differences are in $[1,l-1]$, but we cannot conclude that $l-1\ge n(n-1)/2$ because some of the differences can coincide; namely, we have $a_1-a_2=a_3-a_4$ if and only if exactly one of the following holds: (i) $a_1=a_4$ and $a_2,a_1=a_4,a_3$ is a three-term arithmetic progression; (ii) $a_2=a_3$ and $a_1,a_2=a_3,a_4$ is a three-term arithmetic progression. We associate with every equality of the form $a_1-a_2=a_3-a_4$ the middle term of the corresponding progression.

The crucial observation is that for every element $a\in A$, the integer $2a$ is the middle term of at most one progression: for an equality of the form $a_1+a_2=b_1+b_2=2a$ would contradict the weak Sidon property. Therefore, there are at most $n$ equalities of the form $a_1-a_2=a_3-a_4$. Removing for every such equality the corresponding difference from the count, we get $n(n-1)/2-n\le l-1$; that is, $$ l \ge \frac12(n^2-3n+2) = \frac12(n-1)(n-2)=\binom{n-1}2. $$

$\endgroup$
4
  • 2
    $\begingroup$ Middle terms of these 3-term progressions may take only $n-2$ possible values, not $n$. Therefore $l\geqslant (n-1)(n-2)/2+2$. $\endgroup$ Mar 11, 2020 at 22:33
  • $\begingroup$ Yes, but this lower bound misses Kotzig's Claim by 2. How to get the improvement? $\endgroup$
    – BerndM
    Mar 12, 2020 at 9:51
  • $\begingroup$ @BerndM: see Fedor Petrov's comment. $\endgroup$
    – Seva
    Mar 12, 2020 at 10:44
  • $\begingroup$ @Seva: I saw it, it is still 2 less than Kotzig's claim. $\endgroup$
    – BerndM
    Mar 12, 2020 at 10:50

Your Answer

By clicking “Post Your Answer”, you agree to our terms of service and acknowledge you have read our privacy policy.

Not the answer you're looking for? Browse other questions tagged or ask your own question.